46
$\begingroup$

We know that $$ \sum_{n \le x}\frac{1}{n\ln n} = \ln\ln x + c_1 + O(1/x) $$

where $c_1$ is a constant. Again Mertens' theorem says that the primes $p$ satisfy $$ \sum_{p \le x}\frac{1}{p} = \ln\ln x + M + O(1/\ln x). $$

Thus both these divergent series grow at the same rate. Mertens' theorem was proved without using the prime number theorem, some 25 years before PNT was proved. However from these two examples, we cannot conclude that

$$ \lim_{n \to \infty} \frac{p_n}{n\ln n} = 1 $$ otherwise Mertens' would have been the first to prove PNT. My question is - based on the above two series, what are the technical difficulties that prevent us from reaching the conclusion that $p_n/n\ln n = 1$. There may be counter examples with other series, so such conclusions may not be true in general. However I am not interested in the general case. Instead I am asking only in case of the sequence $1/n\ln n$ and $1/p_n$.

$\endgroup$
6
  • 13
    $\begingroup$ The error in Merten's theorem is $O(1/\log x)$, not $O(1/x)$. I suspect that $O(1/x)$ is not true. $\endgroup$ May 2, 2012 at 12:59
  • 4
    $\begingroup$ Indeed, I suspect that standard techniques (Landau's theorem) prove that the error term in Mertens's theorem is $\Omega(x^{-1/2})$; perhaps Littlewood's technique might even give $\Omega_\pm(x^{-1/2}\log\log\log x)$. $\endgroup$ May 2, 2012 at 17:47
  • 1
    $\begingroup$ @Greg: See my response for a precise estimate. $\endgroup$
    – GH from MO
    May 3, 2012 at 2:38
  • 5
    $\begingroup$ Incidentally, Mertens did give a proof of $\zeta(1+it)=0\implies \zeta(1+2it)=\infty$ based on his 3-4-1 inequality (this was after the proofs of Hadamard and de la Vallee Poussin). So in some sense he could prove PNT. :) $\endgroup$ May 4, 2012 at 7:08
  • 1
    $\begingroup$ @Gjergji : I think Mertens could prove $M(x) = o(x)$ and with the fact that $\zeta(s)$ is meromorphic (for $Re(s) > 1-\epsilon$ is enough) this is enough to imply the PNT $\endgroup$
    – reuns
    Feb 29, 2016 at 23:17

4 Answers 4

83
$\begingroup$

Here is a heuristic that I plan to make into a blog post some day. Suppose that there were only finitely many primes with first digit $9$. Is your estimate good enough to see that?

To be more precise, suppose that there were no primes between $9 \times 10^k$ and $10^{k+1}$ for all sufficiently large $k$. And suppose that the number of primes between $a$ and $b$, for $10^k \leq a < b \leq 9 \times 10^k$ is $\approx \frac{\log 10}{\log 9} \int_{a}^b dt/\log t$ (when $a$ is not too close to $b$). We'll see later where the fraction $\log 10/\log 9$ comes from.

The first thing to note is that this would violate the prime number theorem. In this scenario, we would have $\pi(9 \times 10^k) = \pi(10 \times 10^k)$ for $k$ large. But the prime number theorem says that $$\pi(10\times 10^k) - \pi(9 \times 10^k) \sim \frac{10 \times 10^k}{k \log 10 + \log {10}} - \frac{9 \times 10^k}{k \log 10 + \log {9}} \sim \frac{10^k}{k \log 10} \to \infty.$$ So proving the prime number theorem involves disproving this ridiculous scenario.

Now, let's see that the scenario is consistent with $\sum_{p \leq N} 1/p = \log \log N + M + O(1/\log N)$. The sum over the primes between $10^k$ and $10^{k+1}$ would be roughly $$\frac{\log 10}{\log 9} \int_{10^k}^{9 \times 10^k} \frac{dt}{t \log t} = \frac{\log 10}{\log 9} \left( \log \log (9 \times 10^k) - \log \log 10^k \right)$$ $$=\frac{\log 10}{\log 9} \left( \log( k \log 10+\log 9) - \log (k \log 10) \right) = \frac{\log 10}{\log 9} \left( \log \left( 1+\frac{\log 9}{k \log 10} \right) \right) $$ $$ = \frac{\log 10}{\log 9} \frac{\log 9}{k \log 10} + O(1/k^2)= \frac{1}{k} + O(1/k^2)$$

So $$\sum_{p \leq 10^{n+1}} \frac{1}{p} = \sum_{j=1}^n \left( \frac{1}{j} + O(1/j^2) \right)=$$ $$\log n + B + O(1/n) = \log \log 10^{n+1} + C + O(1/\log 10^{n+1}).$$ Very important exercise left for you: If you redo this computation for $\sum_{p \leq 9 \times 10^k} 1/p$, you get $\log \log (9 \times 10^k) + C + O(1/\log(9 \times 10^k))$ for the same constant $C$. The point is that $\log \log 10^{k+1} - \log \log (9 \times 10^k) = O(1/\log 10^k)$, so this estitmate is consistent with $\sum_{p \leq N} 1/p$ not growing at all between $9 \times 10^k$ and $10 \times 10^k$.

This trick is useful for refuting other simple approaches to the PNT. For example, the "primes hate to start with $9$ scenario" is also consistent with $\sum \log p/p^s = 1/(s-1) + O(1)$ as $s \to 1^{+}$, so that is also not enough to prove PNT.

$\endgroup$
4
  • $\begingroup$ See also my response. $\endgroup$
    – GH from MO
    May 3, 2012 at 2:35
  • 23
    $\begingroup$ Indeed, one can view the known proofs of the prime number theorem as methodically isolating and then eliminating such "conspiracies" among the primes. In particular, by working either with the zeroes of the zeta function or with Selberg's explicit formula, one soon finds that the only conspiracy that can really cause trouble is if primes p hate having $p^{it}$ stray far from the negative axis, for some non-zero real $t$... but then $p^{2it}$ stays too close to the positive real axis, which one can show is in contradiction with the conditional convergence of $n^{2it}/n$. $\endgroup$
    – Terry Tao
    May 4, 2012 at 6:53
  • 15
    $\begingroup$ Or, one can use sieve theory to stop $p^{it}$ clustering near the negative axis too often (this is how the Erdos-Selberg elementary proof goes). $\endgroup$
    – Terry Tao
    May 4, 2012 at 6:54
  • 13
    $\begingroup$ To connect Terry's comment and my post, having no primes between $9 \times 10^k$ and $10 \times 10^k$ is the same as having $p^{2 \pi i/\log 10}$ avoid a certain wedge in $\mathbb{C}$. I just find it easier to think "how would such and such sum behave if no primes started with $9$" then "how would it behave if $\mathrm{arg}(p^{it})$ was never near $\pi$?" $\endgroup$ May 4, 2012 at 17:13
21
$\begingroup$

Let me supplement David Speyer's nice response by elaborating on his original comment and Greg Martin's comment. Let us write $$ \sum_{p\leq x}\frac{1}{p}=\ln\ln x+M+R(x), $$ then we have, using Riemann-Stieltjes integrals, $$ F(x):=\sum_{x < p \leq 2x} \frac{\ln p}{p}=\int_x^{2x}\ln t\ d(\ln\ln t+M) + \int_x^{2x} \ln(t) dR(t) $$ $$ = \int_x^{2x} \frac{dt}{t} + [R(t)\ln t]_x^{2x} - \int_x^{2x} \frac{R(t)}{t} dt = \ln 2 + O( \ln x \sup_{x < t \leq 2x} R(t) ). $$ If $\hat F(s)$ denotes the Mellin transform of $dF(x)$, then with the notation $$ S(x):=\sum_{p \leq x} \frac{\ln p}{p} $$ we have $$ \hat F(s) = \int_{1-}^\infty x^{-s}dS(2x) - \int_{2-}^\infty x^{-s}dS(x) = (2^s-1)\sum_p \frac{\ln p}{p^{s+1}}, \quad \Re s>0. $$ In particular, if $\zeta(s)$ has a zero on $\Re s=\sigma\geq\frac{1}{2}$, then $\hat F(s)$ has a pole on $\Re s=\sigma -1$. Note that on the real segment $s\geq-\frac{1}{2}$, the only singularity of $\hat F(s)$ is $s=-\frac{1}{2}$, coming from the difference between $\sum (\ln p)p^{-s}$ and $\sum\Lambda(n)n^{-s}$. Hence by a well-known principle (see Theorem 11.8 in Bateman-Diamond: Analytic number theory), for a certain $c\in\mathbb{R}$ we infer the two-sided estimates $$ F(x)-\ln 2-c x^{-1/2} = \Omega_\pm(x^{\sigma-1}). $$ This implies, by our initial calculation, $$ R(x) = \Omega(x^{\sigma-1}/\ln x). $$ Here we can take $\sigma=\frac{1}{2}$. In the unlikely case that RH fails, we can choose a larger $\sigma$ and replace $\Omega$ by $\Omega_\pm$. Probably, with more work, we could replace $\Omega$ by $\Omega_\pm$ for $\sigma=\frac{1}{2}$ as well.

Added. Here is a closely related arXiv preprint.

$\endgroup$
11
$\begingroup$

Because the scale is too small in Mertens's theorem, and the prime number theorem as well as the Riemann hypothesis are hidden by the $O(1/\log{X})$ notation.

Indeed, the former amounts to strengthening this term to $o(1/\log{X})$; the latter - to $O(1/\sqrt{X})$.

[Incidentally, one could go for equivalent statements to a still smaller scale. Of course, an estimate like $\sum_{p < X} 1/p^2 = \mathrm{const} + O(1/X)$ does not say anything at all about the primes. But one could, if one wished, express the prime number theorem by elaborating on the $O(1/X)$ term. ]

To elaborate on this a bit, let me go to a slightly bigger scale where the prime number theorem begins to emerge outside of $o(1)$. This is also more natural; indeed, it was how Mertens's theorem was proved.

By partial summation, Mertens's estimate is equivalent to $\sum_{p < X} (\log{p})/(p-1) = \log{X} + O(1)$; or, if one prefers, $\sum_{n < X} \Lambda(n)/n = \log{X} + O(1)$. The prime number theorem however is the statement that the $O(1)$ term converges to a constant: $\sum_{n < X} \Lambda(n)/n = \log{X} - \gamma + o(1)$. Indeed, the related bound $\sum_{n < X} \Big( \frac{1}{n} - \frac{1}{X} \Big) \Lambda(n) = \log{X} - 1 - \gamma + o(1)$, another form of the prime number theorem, is what de la Vallee Poussin actually obtained in his original paper. Here $\gamma = 0.57\ldots$ is Euler's constant, but this is of no importance for us, see the next paragraph. Also the $\log{X}$ logarithmic pole corresponds to the pole of $\zeta(s)$ at $s=1$, whereas the $o(1)$ term expresses there being no zeros with $\mathrm{Re}(s) = 1$. The Riemann hypothesis is the correspondingly stronger bound $O(1/\sqrt{X})$ on the $o(1)$ oscillating term. At this scale, in contrast to $\psi(X) = X + O\big( \sqrt{X} (\log{X})^2 \big)$ or $\pi(X) = \mathrm{Li}(X) + O(\sqrt{X}\log{X})$, a logarithmic factor in addition to the square root is not required, as $\sum_{\rho} 1 / |\rho|^2 < \infty$ over the zeros.

Here finally is how to deduce the more usual form $\psi(X) \sim X$ of the prime number theorem from the refinement $S(X) := \sum_{p < X} \Lambda(n)/n = \log{X} -\gamma + o(1)$ of Mertens's theorem: Summing by parts, $\psi(X) = XS(X) - \int_1^X S(t) \, dt = X(\log{X} - \gamma + o(1)) - \big( \int_1^X \log{t} \, dt - \gamma X + o(X) \big) = X + o(X)$.


Added (December, 2017). I came upon an observation giving also a 'trivial' proof of the reverse elementary implication of the two purely qualitative forms, multiplicative and logarithmic, of the prime number theorem: $\psi(X) \sim X \Leftrightarrow S(X) = \log{X} - \gamma + o(1)$. The following seems to have been missed in the literature on elementary methods which, at this point, seem all to quote a somewhat more involved Tauberian theorem of Axer; cf. section 8.1.1 of Montgomery and Vaughan's book (Multiplicative Number Theory: I) or, for a more general setting, chapter 14 of Diamond and Zhang's recent book on Beurling Generalized Numbers (really this paper of theirs). The simpler argument below also extends easily to number fields, supplying a particularly easy proof of the 'elementary equivalence' of Landau's prime ideal theorem and number field sharp Mertens. Incidentally, as I happen to recall, this addresses a slightly curious point that had come up in the comments to this answer of Eric Naslund. Remembering also my answer here, I figured it may be worth to record the following observation as an addendum to it, sticking for simplicity to the rational case assumed in this question.

A proof of $\psi(X) \sim X \Rightarrow S(X) = \log{X} - \gamma + o(1)$. For simplicity, let me stick to $\mathbb{Q}$. The case of a number field $K$ has the same result with $\gamma$ generalized as the 'Euler-Kronecker invariant' $\gamma_K$.

The key is to observe that the formula $$ X^{-1} \log{X!} = \sum_{n \leq X/T} \frac{\Lambda(n)}{n} + \sum_{m \leq T} \frac{1}{X} \Big( \psi\Big( \frac{X}{m} \Big) - \psi\Big( \frac{X}{T} \Big) \Big) + O(1/T) $$ holds uniformly in the two parameters $X, T \geq 1$, with an absolute implied coefficient. It interpolates between Mertens's estimate (case $T = 1$) and Chebyshev's convolution formula $\log{X!} = \sum_m \psi(X/m)$ (case $T = \infty$). But the general formula also follows, after a moment of reflection, from Chebyshev's argument with the prime factorization of $X!$. Divide the moduli into the ranges $n \leq X/T$ and $n > X/T$. The total contribution of the latter are exactly accounted for by the second sum. For a small modulus $n \leq X/T$, the contribution via the prime factorization is $X^{-1} \lfloor X/n \rfloor \Lambda(n) = \frac{\Lambda(n)}{n} + O\Big(\frac{\Lambda(n)}{X}\Big)$, neglecting the fractional part. The $O(1/T)$ term then comes from summing these for $n \leq X/T$, and using Chebyshev's estimate $\sum_{n \leq Y} \Lambda(n) \ll Y$. (In the number field generalization, the latter estimates extend as lattice point counts.)

Now, by Stirling's asymptotic, the qualitative $\psi(X) \sim X \Rightarrow S(X) = \log{X} - \gamma + o(1)$ implication is immediate from the observed formula upon first letting $X \to \infty$ and then $T \to \infty$.

$\endgroup$
9
$\begingroup$

Thanks to David very interesting remark and GH for the calculation of R(x). I like to present my reasoning why I thought on the Merten's theorem could imply PNT. I would have put it as a comment but due to the length, I am writing as a reply. In all likely hood, I must have committed an error somewhere hence I am reaching the ridiculous conclusion that Merten's theorem implies PNT. I would appreciate if someone tell me where I went wrong.

Let $s(n)$ be a strictly increasing sequence, $s(1) \ge 2$, such that $$ S_s(x) = \sum_{s(r) \le x}\frac{1}{s(r)} = \ln\ln x + C_s + R_s(x). $$ where $C_s$ is a constant that depends only on $s$ and $R_s(x) = o(1)$ is the error term which depends on $s$ and $x$. Let $A=\{[s(1)],[s(2)], [s(3)], \ldots \}$ where [.] denotes the greatest integer function. Let $a(r) = 1$ if $r \in A$ and $a(r) = 0$ if $r \notin A$. Then $N_s(x) = \sum_{r \le x} a(r)$ is the number of elements of $A$ in the interval $(0,x)$. Using Abel's summation formula we obtain $$ \sum_{s(r) \le x}\frac{1}{s(r)} = \frac{N_s(x)}{x} + \int_{s(1)}^{x} \frac{N_s(t)}{t^2}dt = \ln\ln x + C_s + R_s(x) $$ Differentiating under the integral sign with respect to $x$, we obtain $$ \frac{N_s'(x)}{x} - \frac{N_s(x)}{x^2} + \frac{N_s(x)}{x^2} = \frac{1}{x\ln x} + R_s'(x) $$ Simplifying the above equation and integrating both sides, we obtain $$ N_s(x) = \int_{s(1)}^{x}\frac{dt}{\ln t} + \int_{s(1)}^{x} tR_s'(t)dt $$ Without loss of generality, we can define $s(1) =2$ as this will only effect the constant term $C_s$ and not $R_s(x)$. Hence we have $$ N_s(x) = \int_{2}^{x}\frac{dt}{\ln t} + E(x) = Li(x) + E(x). $$ Thus any sequence $s(n)$ satisfying $S_s(x) = \ln\ln x + C_s + R_s(x)$ must have a density function which is asymptotic to the logarithmic integral. We can verify this with the sequences $n\ln n, n\ln n + n\ln\ln n, nH_n$ etc; where $H_n$ is the harmonic number. Also we have found the explicit relationship between the error term in $S_s(x)$ and that of $N_s(x)$ which is $$ E(x) = \int_{2}^{x} tR_s'(t)dt $$

In case of prime numbers, $N_s(x) = \pi (x)$ and Merten's theorem shows that primes satisfy the condition on $S_s(x)$. This gives $$ \pi(x) = \int_{2}^{x}\frac{dt}{\ln t} + E(x). $$

This shows that the fact that $\pi(x) \sim Li(x)$ is a consequence of $S_p(x) \sim \ln\ln x$ and it is not a unique property of primes. In fact it is a common attribute of a general family of sequences which grow at the same asymptotic rate in their dominant term. Primes happen to be just one sequence in this family. What differentiates primes and other members of this family of sequences form each other is difference in their respective error terms $E(x)$ or in terms of $S_s(x)$ the difference in their respective constant terms $C_s$ and their error terms $R_s(x)$. Therefore the correct way to interpret Merten's theorem is that for prime numbers, the constant term in $S_p(x)$ is $$ C_p = M = \gamma + \sum_{k=2}^{\infty} \frac{\mu (k)\ln \zeta(k)}{k} \simeq 0.2614972128 $$ because if it were any other constant, then we know that we are dealing with some other sequence and not the sequence of primes even though the dominant term in asymptotic expansion of the sum of the reciprocal of the sequence would still be $\ln\ln x$ and its density would still be asymptotic to $Li(x)$.

$\endgroup$
4
  • $\begingroup$ First a minor note: $R$ and $N$ aren't actually differentiable functions; they have jump discontinuities at integers. But you can fix that by replacing $R'(x)$ by $(R(x+h)-R(x))/h$ where $h$ is much smaller than $x$ but much larger than $1$. I'll refer to $R'$ below, but what I really mean is "the size of $(R(x+h)-R(x))/h$" for $h$ in the right range. $\endgroup$ May 4, 2012 at 16:50
  • 2
    $\begingroup$ The problem is that $R$ small does not force $R'$ small. In the scenario of my answer, $R(x) = O(1/\log x)$. Between $9 \times 10^k$ and $10 \times 10^k$, the function $R$ changes by $\log \log (10 \times 10^k) - \log \log (9 \times 10^k) \approx c/\log 10^{k+1}$, where $c$ is a constant I don't care to work out. So $R' \approx c/(10^k \log 10^{k+1})$ or $R'(x) \approx c/(x \log x)$. For $10^k < x < 9 \times 10^k$, you get $R'(x) \approx d/(x \log x)$, for a different constant $d$. (continued) $\endgroup$ May 4, 2012 at 16:57
  • $\begingroup$ So, $|R'(x)|$ is of the same order as $d (\log \log x)/dx = 1/(x \log x)$, even though $R(x) << \log \log x$. When you compute $N(t) = \int t d(\log \log t)/dt \cdot dt + \int t R'(t) dt$, the two terms are of the same order of magnitude, and you have no control over $N(t)$. $\endgroup$ May 4, 2012 at 17:07
  • $\begingroup$ @ David, Thank you for the explanation. Now the gap is clear to me. I have accepted your answer as the answer to this question. $\endgroup$ May 6, 2012 at 7:04

Your Answer

By clicking “Post Your Answer”, you agree to our terms of service and acknowledge you have read our privacy policy.

Not the answer you're looking for? Browse other questions tagged or ask your own question.